$$ \newcommand\bm[1]{\boldsymbol{#1}} \renewcommand\limsup{\varlimsup} \renewcommand\liminf{\varliminf} $$

-10 | -1 || 8 / 124 || +1 | +10

[q201110021000]  $n$ を正の整数, $r$ を整数とし, $0\leq r\leq n$ であるとする. このとき, 二項係数 $$ \binom{n}{r} = \begin{cases} \displaystyle\frac{n(n-1)(n-2)\cdots(n-r+1)}{r!}, & \mbox{$1\leq r\leq n$ のとき}, \\ 1, & \mbox{$r=0$ のとき} \end{cases} $$ は整数であることを証明せよ.


[q201110021100]  $n$, $r$ を整数とし, $1\leq r\leq n$ であるとする. このとき, $$ \binom{n}{r-1}+\binom{n}{r} = \binom{n+1}{r} $$ が成り立つことを証明せよ.


[q201110021200]  $p$ を素数, $r$ を整数とし, $1\leq r\leq p-1$ であるとする. このとき, 二項係数 $\displaystyle\binom{p}{r}$ は $p$ で割り切れることを証明せよ.


[q201110021300]  $p$ を素数, $n$ を正の整数, $r$ を整数とし, $1\leq r\leq p^{n}-1$ であるとする. このとき, 二項係数 $\displaystyle\binom{p^n}{r}$ は $p$ で割り切れることを証明せよ.


[q201110021400]  $p$ を素数, $m$, $e$ を正の整数とするとき, \begin{equation} \binom{p^{e}m}{p^{e}}\equiv m\pmod{p} \tag{$*$} \end{equation} が成り立つことを証明せよ.


-10 | -1 || 8 / 124 || +1 | +10

©2003-2011 よしいず